At Westonci.ca, we connect you with experts who provide detailed answers to your most pressing questions. Start exploring now! Explore in-depth answers to your questions from a knowledgeable community of experts across different fields. Join our Q&A platform to connect with experts dedicated to providing accurate answers to your questions in various fields.

inverse laplace of [(1/s^2)-(48/s^5)]

Sagot :

**Refresh page if you see [ tex ]**

I am not familiar with Laplace transforms, so my explanation probably won't help, but given that for two Laplace transform [tex]F(s)[/tex] and [tex]G(s)[/tex], then [tex]\mathcal{L}^{-1}\{aF(s)+bG(s)\} = a\mathcal{L}^{-1}\{F(s)\}+b\mathcal{L}^{-1}\{G(s)\}[/tex]

Given that [tex]\dfrac{1}{s^2} = \dfrac{1!}{s^2}[/tex] and [tex]-\dfrac{48}{s^5} = -2\cdot\dfrac{4!}{s^5}[/tex]

So you have [tex]\mathcal{L}^{-1}\left\{\dfrac{1}{s^2} - 2\cdot\dfrac{4!}{s^5}\right\} = \mathcal{L}^{-1}\left\{\dfrac{1}{s^2}\right\} - 2\mathcal{L}^{-1}\left\{\dfrac{4!}{s^5}\right\}[/tex]

From Table of Laplace Transform, you have [tex]\mathcal{L}\{t^n\} = \dfrac{n!}{s^{n+1}}[/tex] and hence [tex]\mathcal{L}^{-1}\left\{\dfrac{n!}{s^{n+1}}\right\} = t^n[/tex]

So you have [tex]\mathcal{L}^{-1}\left\{\dfrac{1}{s^2}\right\} - 2\mathcal{L}^{-1}\left\{\dfrac{4!}{s^5}\right\} = \boxed{t-2t^4}[/tex].

Hope this helps...
Thanks for using our service. We aim to provide the most accurate answers for all your queries. Visit us again for more insights. We appreciate your time. Please come back anytime for the latest information and answers to your questions. Keep exploring Westonci.ca for more insightful answers to your questions. We're here to help.